Last visit was: 24 Apr 2024, 19:11 It is currently 24 Apr 2024, 19:11

Close
GMAT Club Daily Prep
Thank you for using the timer - this advanced tool can estimate your performance and suggest more practice questions. We have subscribed you to Daily Prep Questions via email.

Customized
for You

we will pick new questions that match your level based on your Timer History

Track
Your Progress

every week, we’ll send you an estimated GMAT score based on your performance

Practice
Pays

we will pick new questions that match your level based on your Timer History
Not interested in getting valuable practice questions and articles delivered to your email? No problem, unsubscribe here.
Close
Request Expert Reply
Confirm Cancel
SORT BY:
Date
Tags:
Show Tags
Hide Tags
User avatar
Retired Moderator
Joined: 06 Jul 2014
Posts: 1010
Own Kudos [?]: 6341 [74]
Given Kudos: 178
Location: Ukraine
Concentration: Entrepreneurship, Technology
GMAT 1: 660 Q48 V33
GMAT 2: 740 Q50 V40
Send PM
Most Helpful Reply
Magoosh GMAT Instructor
Joined: 28 Dec 2011
Posts: 4452
Own Kudos [?]: 28569 [17]
Given Kudos: 130
User avatar
Retired Moderator
Joined: 06 Jul 2014
Posts: 1010
Own Kudos [?]: 6341 [9]
Given Kudos: 178
Location: Ukraine
Concentration: Entrepreneurship, Technology
GMAT 1: 660 Q48 V33
GMAT 2: 740 Q50 V40
Send PM
General Discussion
Director
Director
Joined: 21 May 2013
Posts: 540
Own Kudos [?]: 225 [0]
Given Kudos: 608
Send PM
Re: Egidio's Gym has been in operation for seven years, and offers regular [#permalink]
Harley1980 wrote:
Egidio's Gym has been in operation for seven years, and offers regular weekly weight training and aerobic classes, as well as occasional programs in dance and martial arts. The gym has seen a surge of new members in the past twelve months, despite keeping the same membership fee over the past three years. Of the 450 current members, more than 250 have joined in the past year. Clearly, the membership of Egidio's Gym has risen a significant amount because of its superior facilities.

Which of the following is an assumption on which the argument depends?

A) The vast majority of the 450 current members appreciate the many qualities and services of Egidio's Gym.
B) None of the other gyms in this town saw any increase in membership during the past year.
C) Dance and martial arts have become particularly popular among the general public over the past year.
D) Many of the new members joined because of personal recommendations of those who were already members.
E) Fewer than 250 members ended their membership at Egidio's Gym over the last year.


Harley,
Can you please explain how this can be E. I think it should be A.
Manager
Manager
Joined: 04 May 2014
Status:One Last Shot !!!
Posts: 196
Own Kudos [?]: 607 [6]
Given Kudos: 141
Location: India
Concentration: Marketing, Social Entrepreneurship
GMAT 1: 630 Q44 V32
GMAT 2: 680 Q47 V35
Send PM
Re: Egidio's Gym has been in operation for seven years, and offers regular [#permalink]
6
Kudos
My reasoning lingering in unexplored territories, I request for expert's help here. (mikemcgarry, @Harley1980)

I understand the reasoning behind choosing E over A. And, I question that reasoning.
My claim:
We shouldn't worry about 'Whether the membership rose as much as the author thought it did', but what we need to support with an assumption is the reason for the rise.

Here's why I think so:
Conclusion-
The membership of Egidio's Gym has risen a significant amount because of its superior facilities.

There are two parts to this conclusion (and this is the very reason of this debate):
P1: The membership of Egidio's Gym has risen a significant amount.
P2: The membership of Egidio's Gym has risen because of its superior facilities.

Now, choices A and E play with this nuance. A works with P2 and E with P1 above. E can only be the correct answer if we question P1 i.e. The membership of Egidio's Gym has risen a significant amount. But should we care about it??

As I see it, the conclusion states that the membership rose, as a fact. This info is given to us. However, the reason behind it is questionable.
Q:Why did the membership rise?
Ans: Because of the superior facilities.

This is a leap of faith taken by the author which we need to bridge with an assumption, And NOT that whether the membership rose as much as the author thought it did.

My conclusion:
Though, A talks about 'qualities and services' and not about the 'superior facilities' mentioned in the conclusion, given the above reasoning, it looks better to me than E.

Please correct me...
Thanks!
avatar
Intern
Intern
Joined: 21 Sep 2016
Posts: 2
Own Kudos [?]: [0]
Given Kudos: 0
Send PM
Re: Egidio's Gym has been in operation for seven years, and offers regular [#permalink]
Dear Mike,

I faced this question yesterday in Magoosh platform and I am still confused with this answer.

Let's supposse that at the end of 2015 the members were 449.

The argument says that MORE (sorry i cannot find the bold button) than 250 joined during last year (2016), we can consider 252.

If we go to the statement E and do the negative: MORE than 250 ended their membership at Egidio's Gym over the last year (2016) we can consider 251.

Now we do the math: 449-251+252=450.

So, although E is false the argument still can be valid (a rise in the memberships during 2016).

Maybe the solution could be remove the "more" in the argument in that case E would be the assumption needed.

Thank you for your support.

Posted from my mobile device
Magoosh GMAT Instructor
Joined: 28 Dec 2011
Posts: 4452
Own Kudos [?]: 28569 [2]
Given Kudos: 130
Re: Egidio's Gym has been in operation for seven years, and offers regular [#permalink]
1
Kudos
1
Bookmarks
Expert Reply
Cofocor wrote:
Dear Mike,

I faced this question yesterday in Magoosh platform and I am still confused with this answer.

Let's suppose that at the end of 2015 the members were 449.

The argument says that MORE (sorry i cannot find the bold button) than 250 joined during last year (2016), we can consider 252.

If we go to the statement E and do the negative: MORE than 250 ended their membership at Egidio's Gym over the last year (2016) we can consider 251.

Now we do the math: 449-251+252=450.

So, although E is false the argument still can be valid (a rise in the memberships during 2016).

Maybe the solution could be remove the "more" in the argument in that case E would be the assumption needed.

Thank you for your support.

Dear Cofocor,

I'm happy to respond. :-) It's important to think about the exact wording. Here's what I mean.

Let's follow out your scenario. End of 2015, there were 449 members. In 2016, 252 (more than 250) join, and 251 leave, so membership is now at 450.

Now, suppose the argument is a PR statement made a press conference by some marketing person at Egidio's Gym. This marketing person concludes by saying, "Clearly, the membership of Egidio's Gym has risen a significant amount because of its superior facilities."

Then, the reporter says: "A significant increase? How much did your total membership increase from last year to this year?"

The marketing person would have to say, "Uh, by only one person." Everyone at the press conference would break out into uproarious laughter at this point, because this marketing person would look like an idiot. When we create a scenario that easily makes the person passionately defending the argument look like an idiot, then that's a powerful weakener!

My friend, I think part of what is going on is that you overlooked the implications of the words "significant increase" in the conclusion. Remember that business & academic writing does not use extreme language: the word "significant" is about as powerful and emotionally charged a word as we are likely to see in business writing. It's important not to overlook the punch of such a word.

Also, I would say: don't think of these arguments as mathematical exercises on paper. Think of these in terms of real world characters. A weakening argument is about one person making an argument and someone with a very different agenda trying to prove him wrong. A strengthener is about one person making an argument and another person giving support to the first person. The entire reason the GMAT has a CR section is because the real business world is full of people with very different agendas and very different points of view, and a manager is always in the situation of having to evaluate and respond to a number of arguments about real world issues. The GMAT CR is about real arguments about real situations, exactly like the ones you will see in the business world throughout your career.

Does all this make sense?
Mike :-)
avatar
Intern
Intern
Joined: 21 Sep 2016
Posts: 2
Own Kudos [?]: [0]
Given Kudos: 0
Send PM
Re: Egidio's Gym has been in operation for seven years, and offers regular [#permalink]
Thank you very much for your fast response Mike, point taken.

Just to highlight that the numbers were only examples to support the rational behind the claim.

If in 2015 the members were 100 and in 2016 MORE than 250 join the gym (700) and we take the negative of the statement E: MORE than 250 ended their membership in 2016 (350), the argument still remains valid although the statement E is false (the membership has risen significantly, in this case from 100 to 450: 100+700-350=450). If E is true or false the argument remains valid, cannot be the assumption in which the argument relies.

I think that removing the "MORE" in the argument the solution will be then clearly E.

Thank you Mike.

Posted from my mobile device
Intern
Intern
Joined: 09 Sep 2016
Posts: 4
Own Kudos [?]: [0]
Given Kudos: 0
Send PM
Re: Egidio's Gym has been in operation for seven years, and offers regular [#permalink]
I have a query regarding the ans option E.

If you negate the assumption it should break the conclusion. But in this case, it will strengthen the conclusion. Could someone clarify on this.
Magoosh GMAT Instructor
Joined: 28 Dec 2011
Posts: 4452
Own Kudos [?]: 28569 [2]
Given Kudos: 130
Re: Egidio's Gym has been in operation for seven years, and offers regular [#permalink]
1
Kudos
1
Bookmarks
Expert Reply
Cofocor wrote:
Thank you very much for your fast response Mike, point taken.

Just to highlight that the numbers were only examples to support the rational behind the claim.

If in 2015 the members were 100 and in 2016 MORE than 250 join the gym (700) and we take the negative of the statement E: MORE than 250 ended their membership in 2016 (350), the argument still remains valid although the statement E is false (the membership has risen significantly, in this case from 100 to 450: 100+700-350=450). If E is true or false the argument remains valid, cannot be the assumption in which the argument relies.

I think that removing the "MORE" in the argument the solution will be then clearly E.

Thank you Mike.

Dear Cofocor,

I'm happy to respond. :-) My friend, you are still thinking of this as the way math problems are phrased rather than the way people actually speak in the real world. The GMAT CR, even the ones that mention something numerical, reflect the way real business people talk. It is a profound mistake to read them as if they were math problems.

Here's the prompt again.
Egidio's Gym has been in operation for seven years, and offers regular weekly weight training and aerobic classes, as well as occasional programs in dance and martial arts. The gym has seen a surge of new members in the past twelve months, despite keeping the same membership fee over the past three years. Of the 450 current members, more than 250 have joined in the past year. Clearly, the membership of Egidio's Gym has risen a significant amount because of its superior facilities.

Note that the total is 450, so that's one constraint by which we should abide.

Here is the distinction I want to make.

If a math problem says, "X is greater than 250," then X could be any number on the number line above 250. Even given the integer constraint and the cap of 450, X could be any of the two hundred positive integers from 251 to 450. Moreover, no one of those numbers would be any more likely than any other.

That's how mathematicians talk in math problems. That's not how business people talk in the real world.

This argument is presumably being made by some spokesperson or marketing person for Egidio's Gym. If I want to make the point about how many new members have joined, I am mostly likely going to use a close round number that is very near to the real number. I would say that, from the statement, it's likely that that the number of new members is between 250 and 260, perhaps in the low 260s. Think about it: if 287 were the number of new members just joined, the spokesperson would be wildly undercutting his own argument to say "more than 250"---he would be be more likely to say something like "more than 280" because he is trying to make a point about the size of this group. For this person to underestimate the size of this group would weaken his own argument.

You see, we have to assume that all the arguments in the GMAT CR are made in good faith. Part of what this means is that we have to assume that all the factual data cited, all the premises, are simply true. We also have to assume that the speaker is an intelligent competent person with consistent motivations. We have to assume that the person has a point to make and is not intentionally a self-weakener. If the motivation of this spokesperson is to demonstrate how many new members are joining Egidio's gym, then this person would have absolutely no reason to understate the number he cites, except maybe to round it down to a multiple of ten for simplicity (a lot of people out there are particularly fond of multiples of ten, a phenomenon most perplexing to the true mathematicians.) In this context, "more than 250" would quite likely mean a value below 260.

Now, on the surface, the prompt seems to present a valid argument. More than 50% of the members have joined in the past year. If no one left the gym, and the entire increase could be attributed to new members joining, this scenario would be mean that Egidio's gym grew by more than 100% in a year. That's very impressive growth! That's what the speaker of the argument wants us to believe.

Now, look at the negation of (E)
More than 250 members ended their membership at Egidio's Gym over the last year.

Let's say that 258 joined and 251 left. That's a gain of 7 members, and 7 of 450 is less than a 2% increase. That's pathetic growth. Of course, if the number that left was larger than the number that joined, then there's no increase at all. Anything that demonstrates that the growth was pathetically small or non-existent would obliterate the argument that was predicated on spectacular growth.

I think you are imagining much more mathematical wiggle room because you are not thinking of this in terms of what a real person in the business world would say. That right there is the entire point of the GMAT CR. The GMAT has a CR section so that they can get a sense of how students would handle arguments in the real world.

Does all this make sense?
SanjayaGupta wrote:
I have a query regarding the ans option E.

If you negate the assumption it should break the conclusion. But in this case, it will strengthen the conclusion. Could someone clarify on this.

Dear SanjayaGupta,
I'm happy to respond. :-)

I explained in this post the logic of the question and how the negation of (E) is a powerful weakener. If you do not agree, I will challenge you to explain how you think the negation of (E) strengthens the argument.

Does this make sense?
Mike :-)
Intern
Intern
Joined: 09 Sep 2016
Posts: 4
Own Kudos [?]: [0]
Given Kudos: 0
Send PM
Re: Egidio's Gym has been in operation for seven years, and offers regular [#permalink]
Dear Mike

If you negate the statement in option E it will be like the below one.

Fewer than 250 members didn't end their membership at Egidio's Gym over the last year.

I mean most of the memebers stayed back in the gym over the last year. So, it will be a strenthener.

Please corrct me if I'm wrong.

Thanks you
Sanjay
Magoosh GMAT Instructor
Joined: 28 Dec 2011
Posts: 4452
Own Kudos [?]: 28569 [2]
Given Kudos: 130
Re: Egidio's Gym has been in operation for seven years, and offers regular [#permalink]
2
Kudos
Expert Reply
SanjayaGupta wrote:
Dear Mike

If you negate the statement in option E it will be like the below one.

Fewer than 250 members didn't end their membership at Egidio's Gym over the last year.

I mean most of the memebers stayed back in the gym over the last year. So, it will be a strenthener.

Please corrct me if I'm wrong.

Thanks you
Sanjay

Dear Sanjay,

I'm happy to respond. :-)

My friend, I am not sure exactly how to say this. You have come to a conclusion that would be possible only for an extremely intelligent person who was not a native English speaker. If you apply a purely mathematical way of thinking about the language, one might reach this conclusion, but if one thinks about it the way ordinary English is used, one reaches a very different conclusion. It's this peculiar way that a non-native speaker in any language can make the kinds of mistakes that a native speaker never would have anticipated. Please understand, my friend, this reflects in no way on your intelligence--I am sure you are a quite capable individual! It's just a peculiarity of language learning.

Here's (E):
Fewer than 250 members ended their membership at Egidio's Gym over the last year.

What is the negation of this? You suggested:
Fewer than 250 members did not end their membership at Egidio's Gym over the last year.
To tell you the truth, if someone said this sentence to me, I would have absolutely no idea what that person meant. This is so unusual that it is not only awkward but completely incomprehensible. The only meaning I possibly could assign to this would be that:
Fewer than 250 members chose to continue their membership at Egidio's Gym over the last year.
This would be weird, though, because we would be talking about a different group of people entirely: not group of the people who ended their membership, but another group of members who chose to continue their membership. It's not a simple negation of a statement if we wind up talking about an entirely different group of people.

To negate a sentence, it doesn't always work simply to drop a "not" next to the verb. Especially if comparative words appear, we can't simply negate the verb: instead, we often have to take the opposite of the comparative word. Here, the entire logic of the sentence hinges on the opening comparative word "fewer," so to negate the sentence, we need to take the comparative word with an opposite meaning: "more."
Negation: More than 250 members ended their membership at Egidio's Gym over the last year.

Does all this make sense?
Mike :-)
Manager
Manager
Joined: 23 Jan 2016
Posts: 139
Own Kudos [?]: 81 [0]
Given Kudos: 509
Location: India
GPA: 3.2
Send PM
Re: Egidio's Gym has been in operation for seven years, and offers regular [#permalink]
ok, the conclusion is that gym membership has risen significantly due its superior facilities.

we need to focus on finding an assumption that tells us removes any gap between the premises and the conclusion, or defends the conclusion from breaking down. we cannot target a part of the conclusion that the gym member ship did not rise at all in the first place, because that is not the complete conclusion; the conclusion is whether the gym membership rose DUE to superior facilities.

Can you please explain why E is right?
Magoosh GMAT Instructor
Joined: 28 Dec 2011
Posts: 4452
Own Kudos [?]: 28569 [1]
Given Kudos: 130
Re: Egidio's Gym has been in operation for seven years, and offers regular [#permalink]
1
Kudos
Expert Reply
OreoShake wrote:
ok, the conclusion is that gym membership has risen significantly due its superior facilities.

we need to focus on finding an assumption that tells us removes any gap between the premises and the conclusion, or defends the conclusion from breaking down. we cannot target a part of the conclusion that the gym member ship did not rise at all in the first place, because that is not the complete conclusion; the conclusion is whether the gym membership rose DUE to superior facilities.

Can you please explain why E is right?

Dear OreoShake,

I'm happy to respond. :-)

My friend, with all due respect, I believe you are thinking too rigidly, too formulaically, about the GMAT CR. This is precisely what is challenging about the GMAT CR--each argument is a little different, and there's something unique about that situation that is crucial for completing the CR task.

If the gym has no rise in membership at all, then this devastates any argument that the rise in membership was due to any particular factor. If there's no rise, then there is no rise due to a specific reason. The existence of a rise is not "part" of the conclusion, a part separable from the whole.

In some sense, it is an assumption of every argument on the GMAT that life on earth will continue after today. This would not be relevant only to "part" of any argument: it would destroy and render meaningless the whole of almost every argument. Of course, no GMAT CR argument is going to challenge that assumption, so it's never going to be relevant to the answer of a question, but the point is, there are dozens of ways to support or disrupt the logical flow of an argument.

There's a big difference between a side-by-side relationship vs. a foundational relationship. In some arguments, two elements are side by side: A could exist without B, or B could exist without A. Even then, if the conclusion is about A & B, it may be that an assumption is about only A or only B. In this case, though, we have a foundational relationship: B depends on A any only happens if A happens. In this case, it's perfectly appropriate that the assumption treat only A, because without A, we don't have B.

Does all this make sense?
Mike :-)
Manager
Manager
Joined: 23 Jan 2016
Posts: 139
Own Kudos [?]: 81 [0]
Given Kudos: 509
Location: India
GPA: 3.2
Send PM
Re: Egidio's Gym has been in operation for seven years, and offers regular [#permalink]
mikemcgarry wrote:
OreoShake wrote:
ok, the conclusion is that gym membership has risen significantly due its superior facilities.

we need to focus on finding an assumption that tells us removes any gap between the premises and the conclusion, or defends the conclusion from breaking down. we cannot target a part of the conclusion that the gym member ship did not rise at all in the first place, because that is not the complete conclusion; the conclusion is whether the gym membership rose DUE to superior facilities.

Can you please explain why E is right?

Dear OreoShake,

I'm happy to respond. :-)

My friend, with all due respect, I believe you are thinking too rigidly, too formulaically, about the GMAT CR. This is precisely what is challenging about the GMAT CR--each argument is a little different, and there's something unique about that situation that is crucial for completing the CR task.

If the gym has no rise in membership at all, then this devastates any argument that the rise in membership was due to any particular factor. If there's no rise, then there is no rise due to a specific reason. The existence of a rise is not "part" of the conclusion, a part separable from the whole.

In some sense, it is an assumption of every argument on the GMAT that life on earth will continue after today. This would not be relevant only to "part" of any argument: it would destroy and render meaningless the whole of almost every argument. Of course, no GMAT CR argument is going to challenge that assumption, so it's never going to be relevant to the answer of a question, but the point is, there are dozens of ways to support or disrupt the logical flow of an argument.

There's a big difference between a side-by-side relationship vs. a foundational relationship. In some arguments, two elements are side by side: A could exist without B, or B could exist without A. Even then, if the conclusion is about A & B, it may be that an assumption is about only A or only B. In this case, though, we have a foundational relationship: B depends on A any only happens if A happens. In this case, it's perfectly appropriate that the assumption treat only A, because without A, we don't have B.

Does all this make sense?
Mike :-)


So what you are saying is if the a premise of the argument has incomplete information or is invalid in the first place, it wouldnt even matter whether the conclusion happened because of xyz reasons because, there will be no conclusion to begin with if premise is proven incomplete or incorrect? Please let me know if my understanding of what you said is wrong. I have conditioned myself to focus on the conclusion and its premise, generally indicated through markers such as the words 'clearly' and 'because' in the last sentence. in this question, the conclusion may be the complete last sentence.

Thanks Mike
Magoosh GMAT Instructor
Joined: 28 Dec 2011
Posts: 4452
Own Kudos [?]: 28569 [1]
Given Kudos: 130
Re: Egidio's Gym has been in operation for seven years, and offers regular [#permalink]
1
Bookmarks
Expert Reply
OreoShake wrote:
So what you are saying is if the a premise of the argument has incomplete information or is invalid in the first place, it wouldnt even matter whether the conclusion happened because of xyz reasons because, there will be no conclusion to begin with if premise is proven incomplete or incorrect? Please let me know if my understanding of what you said is wrong. I have conditioned myself to focus on the conclusion and its premise, generally indicated through markers such as the words 'clearly' and 'because' in the last sentence. in this question, the conclusion may be the complete last sentence.

Thanks Mike

Dear OreoShake,

I'm happy to respond. :-)

Actually, that's not really what I am saying. I will try again.

The Zen recluse and poet Han Shan (9th century) said: "No path goes all the way." In this simple sentence, the poet says something deep about human life and something deep about GMAT Critical Reasoning.

On the CR questions, the GMAT is assessing flexible critical thinking. If you have any fixed approach to analyzing the arguments, the GMAT will punish that. The GMAT excels a producing a wide variety of arguments: ultimately, each argument is different and needs to be appreciated in its own terms.

I would say there is absolutely no such thing as a premise with "complete information." That is a chimera. Completely information would involve all life on earth continuing, the US government surviving, the US monetary systems continuing, human nature remaining the same, etc. etc. Everything is connected to everything else, so no premise could ever have "complete information."

Sometimes, we weaken an argument by finding a flaw in the premises. Sometimes, we weaken an argument by attacking the assumption. Sometimes, we weaken an argument by showing what illogical further conclusion it implies. Often, the task on a CR question is intimately related to the particularity of the situation--this unique relationship makes it impossible to generalize fully the task on GMAT CR. The GMAT wants to know whether you have flexible critical thinking, and no formula or pattern or shortcut will produce that.

The best way to practice for GMAT CR is not to formulate some list of rules. The best way to practice is to read arguments, real-world arguments. Read the business news, including the Economist magazine, the Financial Times, and Bloomberg Businessweek. Read about the real issues and real arguments that arise in the modern business world, and think about those arguments. Each one is situation is different, and a good manager is someone who can assess, relatively quickly, what each new unique situation demands.

Does all this make sense?
Mike :-)
Retired Moderator
Joined: 23 Sep 2015
Posts: 1267
Own Kudos [?]: 5650 [1]
Given Kudos: 416
Send PM
Re: Egidio's Gym has been in operation for seven years, and offers regular [#permalink]
1
Kudos

Official Explanation


The credited answer is (E). A significant piece of evidence in the argument is that 250 new members joined over the past year: this is taken to indicate a swell in membership. For an assumption question, we use the Negation Test. Take the negative of (E). If more than 250 members ended their membership over the past year, then the loss would be more than the gain: this would indicate the membership actually declined over the past year! That would devastate the argument. Since negating (E) devastates the argument, we know it is an assumption.

Choice (A) is suggestive, but not an assumption. Let's negate this. Suppose just about all of the 200 previous members don't have a particular high opinion of Egidio's Gym: perhaps they are creatures of habit settling for the status quo. Suppose some number, say 70, chose Egidio's for cost or convenience or some similar external reason. Suppose the other 180 new members love Egidio's facilities. Then, it would be simultaneously true that not a vast majority of the 450 current members appreciates the many qualities and services of Egidio's Gym and yet that a significant fraction of the new members does appreciate it, supporting the idea that Egidio's superior facilities drives new membership. We can negate this and still imagine the argument succeeding, so this is not an assumption. Choice (A) is incorrect.

Choice (B) is also suggestive, but not an assumption. Let's negate this. True, if every gym in town saw a huge surge in membership, then it would be hard to explain anything unique to Egidio's Gym because of the membership increase. BUT, the negation of (B) simply would be that some other gyms saw some increase. Suppose, say, 2/3 of the other gyms so no increase, and 1/3 of them saw an under-10% increase. Then (B) would be false, but the increase of Egidio's would still stand out as exceptional, and the argument could still stand. We can negate this and still imagine the argument succeeding, so this is not an assumption. Choice (B) is incorrect.

Dance and martial arts may or may not be popular in the general public, but this doesn't change the fact that Egidio's Gym saw 250 new members, so either way, it doesn't influence the core of the argument. Choice (C) is incorrect.

It doesn't matter whether new members were impressed on their own, say, during free tours of the facilities, or whether they had personal recommendations. The fact remains: 250 new members. Whether this is true or false doesn't influence the core of the argument. Choice (D) is incorrect.
Director
Director
Joined: 16 Jun 2021
Posts: 994
Own Kudos [?]: 183 [0]
Given Kudos: 309
Send PM
Re: Egidio's Gym has been in operation for seven years, and offers regular [#permalink]
The key to cracking the question is to bring counterargumnet and check whether it stands to the test after completing the scrutiny the only one standing was IMO E

A) The vast majority of the 450 current members appreciate the many qualities and services of Egidio's Gym.
We don't know whether it is due to increased quality or some other factor example everyone joining the gym might be exempted fron the tax just an example to prove that we don't know the exact reasons

B) None of the other gyms in this town saw any increase in membership during the past year.
We have absolutely no clue as of what happend to them

C) Dance and martial arts have become particularly popular among the general public over the past year.
Here too we are not in a position to comment we can use a similar example as and opt of the option

D) Many of the new members joined because of personal recommendations of those who were already members.
Here too no evidience is provided by the argument

E) Fewer than 250 members ended their membership at Egidio's Gym over the last year
Yes this can be clearly infered from the same
Hence IMO E
Manager
Manager
Joined: 10 Jul 2021
Posts: 233
Own Kudos [?]: 48 [0]
Given Kudos: 29
Send PM
Re: Egidio's Gym has been in operation for seven years, and offers regular [#permalink]
Please note the SC issue here:-

Clearly, the membership of Egidio's Gym has risen a significant amount because of its superior facilities.

Should it be significant number or amount.

Posted from my mobile device
User avatar
Non-Human User
Joined: 01 Oct 2013
Posts: 17213
Own Kudos [?]: 848 [0]
Given Kudos: 0
Send PM
Re: Egidio's Gym has been in operation for seven years, and offers regular [#permalink]
Hello from the GMAT Club VerbalBot!

Thanks to another GMAT Club member, I have just discovered this valuable topic, yet it had no discussion for over a year. I am now bumping it up - doing my job. I think you may find it valuable (esp those replies with Kudos).

Want to see all other topics I dig out? Follow me (click follow button on profile). You will receive a summary of all topics I bump in your profile area as well as via email.
GMAT Club Bot
Re: Egidio's Gym has been in operation for seven years, and offers regular [#permalink]
Moderators:
GMAT Club Verbal Expert
6920 posts
GMAT Club Verbal Expert
238 posts
CR Forum Moderator
832 posts

Powered by phpBB © phpBB Group | Emoji artwork provided by EmojiOne